Vector Cross Product With Its Curl

Click For Summary
The discussion focuses on deriving the vector cross product and its relation to curl using index notation. The user is attempting to equate the left-hand side (LHS) to the right-hand side (RHS) of a given problem but is confused about the origin of a factor of 1/2. The explanation highlights that the gradient of A squared results in two terms, leading to the necessity of dividing by two to account for the duplication. This results in the appearance of the term ∇(A²/2) in the identity being proven. Understanding this step is crucial for resolving the user's confusion regarding the derivation.
John Delaney
Messages
3
Reaction score
1
Homework Statement
Prove A X (∇ X A) = ∇(A²/2) - A · ∇A
Relevant Equations
εijk εlmk = δil δjm - δim δjl
Starting with LHS:

i εijk Aj (∇xA)k

i εijk εlmk Aj (d/dxl) Am

il δjm - δim δjl) Aj (d/dxl) Ami

δil δjm Aj (d/dxl) Ami - δim δjl Aj (d/dxl) Ami

Aj (d/dxi) Aji - Aj (d/dxj) Aii

At this point, the LHS should equal the RHS in the problem statement, but I have no clue where the 1/2 comes from...been trying to figure it out for the past few hours at this point. Rather, I get this and I'm not sure where my lapse in understanding is (I'm relatively new to index notation):

∇(A²) - A · ∇A
 
  • Like
Likes Delta2
Physics news on Phys.org
What is the derivative of f(x)=x2/2?
Your expression here is analogous.
 
Notice that ##A^2 = A \cdot A##. When the gradient operator is applied to this term, you get two terms looking like this: ##A(\nabla A)##. In your initial work, you only have one of these terms, so to account for the duplicate, you must divide ##\nabla A^2## by two, which results in the term ##\nabla \frac{A^2}{2}## appearing in the identity you are attempting to prove.
 
  • Like
Likes Delta2
Question: A clock's minute hand has length 4 and its hour hand has length 3. What is the distance between the tips at the moment when it is increasing most rapidly?(Putnam Exam Question) Answer: Making assumption that both the hands moves at constant angular velocities, the answer is ## \sqrt{7} .## But don't you think this assumption is somewhat doubtful and wrong?

Similar threads

  • · Replies 11 ·
Replies
11
Views
4K
  • · Replies 11 ·
Replies
11
Views
8K
  • · Replies 10 ·
Replies
10
Views
6K
  • · Replies 2 ·
Replies
2
Views
2K
Replies
4
Views
2K
Replies
14
Views
12K
  • · Replies 1 ·
Replies
1
Views
1K
Replies
6
Views
8K
Replies
21
Views
3K
  • · Replies 2 ·
Replies
2
Views
4K